You are on page 1of 4

Multilinear Algebra & Canonical Forms

Homework #1
Cristian Camilo Chica Castao

Problems 8, 10, 12, 21 from exercises section 10.1 Dummit and Foote.
In these exercises R is a ring with 1 and M is a left R-module.
8. An element m of the R-module M is called a torsion element if rm = 0 for some nonzero element r

in R. The set of torsion elements is denoted Tor(M)={m M | rm = 0 for some nonzero r R}.
(a) Prove that if R is an integral domain then Tor(M) is a submodule of M (called the
torsion submodule of M).

Proof:

We remember rst that if R is an integral domain then R is a commutative ring with 1 and
no zero-divisors. By propisition 1 (Mentioned in class) Tor(M) is a submodule of M if satises: a)
T or(M ) 6= b) for all r in R, and x,y in T or(M ), x + ry T or(M ). Since (M, +) is an abelian
group then 0 M and for every r in R, r0 = 0 then 0 T or(M ), hence a) is veried. Now we take x
and y in T or(M ) then there exist some p, q in R 0 such thatpx = 0 and qy = 0. Let s = pq then
s 6= 0 since p,q are nonzero elements of R which doesn't have zero-divisor. Because R is a commutative
ring and using the properties given in the denition of a R-module then s(x + ry) = sx + s(ry) =
(pq)x + (sr)y = (qp)x + (pqr)y = q(px) + (pr)qy = q0 + (pr)0 = 0, hence b) is veried. Tor(M) is a
submodule.
(b) Give an example of a ring R and R-module M such that Tor(M) is not a submodule.

Proof:

We take R = Z6 then M = Z6 is a Z6 - module. Now 2 and 3 belongs to T or(M ) since


2 3 = 6mod6 = 0 then if T or(Z6 ) was a submodule of Z6 then it has to be an abelian group itself,
hence 2 + 3 = 5 T or(Z6 ) i.e., there is an r in Z6 {0} such that r5 = 0 but this can't be because 5
is a unit in Z6 .
(c) If R has zero divisors show that every nonzero R-module has nonzero torsion elements.

Proof: Let M a nonzero R-module then there is an element m M {0} and a zero divisor a R{0}
we have that rm T or(M ) since there is a b R {0} such that ab = 0 and a(bm) = (ab)m = 0.

10. If I is a right ideal of R, the annihilator of I in M is dened to be {m M | am = 0 for all a I}.


Prove that the annihilator of I in M is a submodule of M.

Proof:

By proposition 1 mentioned above, let see that the set A:={m M | am = 0 for all a I}
also satises a) A 6= and b) for all r in R, and x,y in A, x + ry A. First as (M, +) is an abelian
group then 0 M then a0 = 0 for all a in I, hence 0 A proving that a) holds. Second, let r in R
and x and y in A then a(x + ry) = ax + a(ry) = 0 + (ar)y = 0 + 0 = 0. ax = 0 by denition of A
and (ar)y = 0 because ar I since I is a right ideal of R. Hence b) is also satised. Therefore A is a
submodule of M.

12. In the notation of the preceding exercises prove the following facts about annihilators.

(a) Let N be a submodule of M and let I be its annihilator in R. Prove that the annihilator
of I in M contains N . Give an example where the annihilator of I in M does not equal
N.
1

Proof:

If I is the annihilator of N in R is dened as I ={r R | rn = 0 for all n N } from the


exercise 9. I is a two-sided ideal of R, then the annihilator A:={m M | am = 0 for all a I} of I
in M is well dened. Now let see that N A. We take n in N now a n = 0 for all a I , because a
belongs to the annihilator of N in R, i.e., a n = 0 for all n in N in particular for n . This mean n A
and we get what we wanted. For the example consider M = Z6 as Z3 -module and let N = {0, 2, 4} be
a Z3 -submodule, because N is a subgroup of Z6 which is closed under multiplication by the elements of
Z3 as the next simple calculation show us: 0n = 0, 1n = n, 2 0 = 0, 2 2 = 4, 2 4 = 8 = 2 for all n and
all of them belongs to N . Now the annihilator of N in Z3 is I={r Z3 |rn = 0 for all n in N }={0}
and the annihilator of I in Z6 is A={m Z6 | am = 0 for all a {0}}= Z6 . Clearly N ( A = Z6 .
(b) Let I be a right ideal of R and let N be its annihilator in M . Prove that the annihilator
of N in R contains I . Give an example where the annihilator of N in R does not equal
I.

Proof:

By denition N :={m M | am = 0 for all a I} and by exercise 10. N is a R-submodule


of M then the annihilator A:={r R | rn = 0 for all n N } of N in R is well dened. By exercise
9. A is two-sided ideal of R. Let see that I A. We take an element in I say x, then xn = 0 for
all nN because if n belongs to N , by denition of the annihilator an = 0 for all a I in particular
for a = x, hence x A and I A. For the example consider Z3 as a Z9 -module and take the ideal
I of Z9 as I = {0, 3, 6}. The annihilator N of I in Z3 , as a simple calculation show us, is dened as
N :={m Z3 | am = 0 for all a I} = {0} and the annihilator of N in R is A:={r Z9 | rn = 0 for
all n N } ={r Z9 | rn = 0 for all n {0}} = Z9 . Finally I = {0, 3, 6} ( A = Z9 .

21. Let n Z

+
, n 1 and let R be the ring of n n matrices with entries from a eld F . Let M be
the set of n n matrices with arbitrary elements of F in the rst column and zeros elsewhere.
Show that M is a submodule of R when R is considered as a left module over itself, but M is
not a submodule of R when R is considered as a right R-module.

a11 a12 a1n


a11 0 0
a21

a21

We dened R = {
| aij F } and M = { .
..

| ai1
.
.
..
..
.

..

an1 an2 ann


an1 0 0
F } Let see that M is a R-submodule of R. By proposition 1 mentioned above let see that M satises
a) M 6= and b) for all C in R, and A,B in M , A + CB M , where A, B and C are n n matrices.
Since F is a eld then 0 F and null matrix belongs to M , hence a) is veried. Now let C in R, and
A,B in M we have that:

a11 0 0
c11 c12 c1n
b11 0 0
a21
c21
b21

A + CB = .
+ .

= A+
.
.
.
.
..
..
..
..
..
..

an1 0

0
cn1 cn2 cnn
n
bn1 0 0
n
P
P
k=1 c1k bk1 0 0 a11 + k=1 c1k bk1 0 0
n

n
P
P

c
b
a
+
c
b
k=1 2k k1
21 k=1 2k k1

.
.
.
.
..
..
..
..

n
P
P

cnk bk1 0 0
an1 +
cnk bk1 0 0

Proof:

k=1

k=1

then A + CB M and b) is also satised. Hence M is a R-submodule of R. Now we want to show


that M is not a submodule of R when R is considered as a right R-module. We only have to show
that M isn't closed under right multiplication by the elements of R. Let A in M and B in R be two
n n matrices, let see that AB
/ M.
2

a11
a21

AB = .
..
an1

..

b11
b21

..
.
0
bn1

b12

..
bn2

b1n

bnn

a11 b11
a21 b11

=
..

.
an1 b11

a11 b12

..
an1 b12

a11 b1n

/ M.

an1 b1n

Problems 6, 12, from exercises section 10.2 Dummit and Foote.


6. Prove that Hom (Z/nZ, Z/mZ) = Z/(n, m)Z.
Z

First let us simplify the notation, since Z/nZ


= Zn , Z/mZ
= Zm and let d = (n, m) then Z/(n, m)Z
=
Zd , hence the problem becomes in HomZ (Zn , Zm )
= Zd . We know that every homomorphism from
Zn to Zm is determined by which k in Zm goes (1) . Since (0) = (n) = (n 1) = n(1) =
(n k)modm = 0 then |k| must divide n, and because (1) = k Zm |k| must divide m too, and by
denition of d = (n, m), |k| must divide d . Therefore, every function HomZ (Zn , Zm ) must satisfy
that |(1)| = |k| divide d, and because this there is a bijection between the elements in HomZ (Zn , Zm )
and the elements which his order divide d and are minor or equal than d , more precisely Zd . The
previous armation show that |Zd | = d = |HomZ (Zn , Zm )| . Now let see that HomZ (Zn , Zm ) is a
clyclic gruop. We take and element t in Zd such that hti = Zd then for t there is an homomorphism
t (1) such that |t (1)| = |t| = d but |HomZ (Zn , Zm )| = d = |t (1)| then ht (1)i = HomZ (Zn , Zm ).
Finally by fundamental theorem of cyclic groups there is only one cyclic group of order d which is Zd .

12. Let I be a left ideal of R and let n be a positive integer. Prove R /IR
n

times). Where IR is dened as in exercise 5 of Section 10.1.


n

= R/IR R/IR (n

Proof: We start
describing the sets involve in the proof:

1) Rn = { x P
= (x1 , ..., xn ) | xi R},

2) IRn = {
ai
xi : ai I,
xi Rn },
f inite

3) Rn /IRn P
= {
x + IRn :
x Rn },
4) IR = {
ai xi : ai I, xi R},
f inite

5) R/IR = {x+IR : x R}
6) (R/IR)n := R/IR R/IR = {(x1 + IR, ..., xn + IR) : xi R}.
We add some comments about the sets 3) and 5), they are well dened since Rn and R are rings
and the divisors IRn , IR are ideals in each ring respectively. Now the prove consist in show and
isomorphism between two R-modules, for which we consider the next function: : Rn /IRn (R/IR)n

such that (
x + IRn ) = ((x1 , ..., xn ) + IRn ) = (x1 + IR, ..., xn + IR). Let see that is a well dened
function. i) Independence of the representative elements of the class and injectivity:

Suppose
x + IRn =
y + IRn
x
y IRn
x
y =

ai I,
zi Rn
x
y =

ai (z1 , ..., zn )i
x
y =

ai zi
f inite
a1 (z11 , ..., zn1 ) +

f inite
f.s.m>0

amP
(z1m , ..., znm ) P

x
y = (a1 z11 + P
+ am z1m , ..., a1 zn1 + + am znm )
x
y =
i
i
i
i
(
ai z1 , ...,
ai zn ) xk yk =
ai zk ai I, zk R; k = 1, ..., n
f inite

f inite

f inite

xk yk IR; k = 1, ..., n xk + IR = yk + IR; k = 1, ..., n (x1 + IR, ..., xn + IR) =

(y1 + IR, ..., yn + IR) ((x1 , ..., xn ) + IRn ) = ((y1 , ..., yn ) + IRn ) (
x + IRn ) =

n
( y + IR ).

ii) is a homomorphism:

Let
x + IRn ,
y + IRn Rn /IRn : ((
x + IRn ) + (
y + IRn )) = ((
x +
y ) + IRn ) =
((x1 + y1 ) + IR, ..., (xn + yn ) + IR) = ((x1 + IR) + (y1 + IR), ..., (xn + IR) + (yn + IR)) =

(x1 + IR, ..., xn + IR) + (y1 + IR, ..., yn + IR) = (


x + IRn ) + (
y + IRn ).

n
n
Let r R: (r( x + IR )) = (r x + IR ) = ((rx1 , ..., rxn ) + IRn ) = (rx1 +

IR, ..., rxn + IR) = (r(x1 + IR), ..., r(xn + IR)) = r(x1 + IR, ..., xn + IR) = r((
x + IRn )).

ii) is surjective: Let (x1 + IR, ..., xn + IR) be an element in (R/IR)n we want to see him as the
image under of an element in Rn /IRn , is enough to take the element (x1 , ..., xn ) + IRn .

You might also like